Difference between revisions of "2019 AMC 10A Problems/Problem 4"

(Solution)
(merge solutions 1 and 3 b/c they are the same, just worded differently)
(19 intermediate revisions by 11 users not shown)
Line 3: Line 3:
 
==Problem==
 
==Problem==
  
A box contains <math>28</math> red balls, <math>20</math> green balls, <math>19</math> yellow balls, <math>13</math> blue balls, <math>11</math> white balls, and <math>9</math> black balls. What is the minimum number of balls that must be drawn from the box without replacement to guarantee that at least <math>15</math> balls of a single color will be drawn<math>?</math>
+
A box contains <math>28</math> red balls, <math>20</math> green balls, <math>19</math> yellow balls, <math>13</math> blue balls, <math>11</math> white balls, and <math>9</math> black balls. What is the minimum number of balls that must be drawn from the box without replacement to guarantee that at least <math>15</math> balls of a single color will be drawn?
  
 
<math>\textbf{(A) } 75 \qquad\textbf{(B) } 76 \qquad\textbf{(C) } 79 \qquad\textbf{(D) } 84 \qquad\textbf{(E) } 91</math>
 
<math>\textbf{(A) } 75 \qquad\textbf{(B) } 76 \qquad\textbf{(C) } 79 \qquad\textbf{(D) } 84 \qquad\textbf{(E) } 91</math>
  
 
==Solution==
 
==Solution==
We want to maximize the number terms. Since we are not limited by positive integers, we know the answer can be greater than 9. We see that the sum of integers from <math>-n</math> to <math>n</math> is <math>0</math>, so we can choose <math>n</math> to be 44. Then the only number to add left is <math>45</math>.
+
 
Therefore the number of terms is <math>44+44+1+1 = \boxed{90}</math>
+
We can find the maximum number of balls that can be drawn while getting <math><15</math> of each color by applying the [[pigeonhole principle]].  
 +
Namely, we can draw up to <math>14</math> red balls, <math>14</math> green balls, <math>14</math> yellow balls, <math>13</math> blue balls, <math>11</math> white balls, and <math>9</math> black balls, for a total of <math>75</math> balls, without drawing <math>15</math> balls of any one color. Drawing one more ball guarantees that we will get <math>15</math> balls of one color — either red, green, or yellow. Thus, the answer is <math>75 + 1 = \boxed{\textbf{(B) } 76}</math>.
 +
 
 +
==Video Solution 1==
 +
 
 +
https://youtu.be/givTTqH8Cqo
 +
 
 +
Education, The Study of Everything
 +
 
 +
== Video Solution 2 ==
 +
https://youtu.be/8WrdYLw9_ns?t=23
 +
 
 +
~ pi_is_3.14
 +
 
 +
==Video Solution 3==
 +
https://youtu.be/2HmS3n1b4SI
 +
 
 +
~savannahsolver
  
 
==See Also==
 
==See Also==

Revision as of 17:33, 3 August 2021

The following problem is from both the 2019 AMC 10A #4 and 2019 AMC 12A #3, so both problems redirect to this page.

Problem

A box contains $28$ red balls, $20$ green balls, $19$ yellow balls, $13$ blue balls, $11$ white balls, and $9$ black balls. What is the minimum number of balls that must be drawn from the box without replacement to guarantee that at least $15$ balls of a single color will be drawn?

$\textbf{(A) } 75 \qquad\textbf{(B) } 76 \qquad\textbf{(C) } 79 \qquad\textbf{(D) } 84 \qquad\textbf{(E) } 91$

Solution

We can find the maximum number of balls that can be drawn while getting $<15$ of each color by applying the pigeonhole principle. Namely, we can draw up to $14$ red balls, $14$ green balls, $14$ yellow balls, $13$ blue balls, $11$ white balls, and $9$ black balls, for a total of $75$ balls, without drawing $15$ balls of any one color. Drawing one more ball guarantees that we will get $15$ balls of one color — either red, green, or yellow. Thus, the answer is $75 + 1 = \boxed{\textbf{(B) } 76}$.

Video Solution 1

https://youtu.be/givTTqH8Cqo

Education, The Study of Everything

Video Solution 2

https://youtu.be/8WrdYLw9_ns?t=23

~ pi_is_3.14

Video Solution 3

https://youtu.be/2HmS3n1b4SI

~savannahsolver

See Also

2019 AMC 10A (ProblemsAnswer KeyResources)
Preceded by
Problem 3
Followed by
Problem 5
1 2 3 4 5 6 7 8 9 10 11 12 13 14 15 16 17 18 19 20 21 22 23 24 25
All AMC 10 Problems and Solutions
2019 AMC 12A (ProblemsAnswer KeyResources)
Preceded by
Problem 2
Followed by
Problem 4
1 2 3 4 5 6 7 8 9 10 11 12 13 14 15 16 17 18 19 20 21 22 23 24 25
All AMC 12 Problems and Solutions

The problems on this page are copyrighted by the Mathematical Association of America's American Mathematics Competitions. AMC logo.png